0 Daumen
138 Aufrufe

Aufgabe:

Bestimmen sie den Grenzwert der Folge : (n2 +1) / (\( \sqrt{3n^4-8n+13} \) )


Problem/Ansatz:

So bin ich vorgegangen:

Zuerst habe ich den Bruch quadriert → (n^4 + 2n^2 +1) / (3n^4 -8n +13)

Dann habe ich n^4 ausgeklammert vom Zähler und Nenner → n^4 *(1 +2/n^2 +1/n^4) / n^4 *(3- 8/n^3 +13/n^4)

und zum Schluss habe ich den Grenzwert 1/3 erhalten da 2/n^2 +1/n^4  und 8/n^3 +13/n^4 für n gegen unendlich  0 ist .


Die Lösung ist aber 1/√3

was habe ich falsch gemacht ?

Danke im Voraus

Avatar von

3 Antworten

+3 Daumen
 
Beste Antwort

Aloha :)

Wenn du den Ausdruck für die Folgenglieder etwas umformst:$$a_n=\frac{n^2+1}{\sqrt{3n^4-8n+13}}=\frac{n^2+1}{\sqrt{n^4\cdot\left(3-\frac{8}{n^3}+\frac{13}{n^4}\right)}}=\frac{n^2+1}{\sqrt{n^4}\cdot\sqrt{3-\frac{8}{n^3}+\frac{13}{n^4}}}$$$$\phantom{a_n}=\frac{n^2+1}{n^2\cdot\sqrt{3-\frac{8}{n^3}+\frac{13}{n^4}}}=\frac{\pink{\frac{1}{n^2}}\left(n^2+1\right)}{\pink{\frac{1}{n^2}}\left(n^2\cdot\sqrt{3-\frac{8}{n^3}+\frac{13}{n^4}}\right)}=\frac{1+\frac{1}{n^2}}{\sqrt{3-\frac{8}{n^3}+\frac{13}{n^4}}}$$

kannst du den Grenzwert ablesen:$$\lim\limits_{n\to\infty}(a_n)=\frac{1+0}{\sqrt{3-0+0}}=\frac{1}{\sqrt3}$$

Das passt zu deinem Ergebnis, denn du hast ja die Folgenglieder quadriert, also den Grenzwert von \((a_n^2)\) bestimmt, und als Ergebnis \(\frac13\) erhalten.

Das Quadrieren von Folgendgliedern kann bei der Grenzwertbestimmung aber nach hinten losgehen, etwa wenn die Folgenglieder ihr Vorzeichen wechseln. Daher solltest du diese Methode nicht allgemein verwenden.

Avatar von 148 k 🚀

Ästhetisch perfekt!

Kein Zwischenschritt weggelassen.

Ein echter Tschaka. :)

0 Daumen

Klammere n^2 aus:

Zähler: n^2*(1+1/n^2)

Nenner:

-> n^2*√(3-8/n^3+13/n^4)

Mit n^2 kürzen:

-> 1/√3

Der Rest unter der Wurzel geht gg. 0.

Avatar von 37 k
0 Daumen

Du hast alles richtig gemacht, aber da du die Folge quadriert hast, erhältst du auch das Quadrat des Grenzwerts. Das kannst du durch Wurzelziehen wieder rückgängig machen und dann passt alles.

Avatar von 6,0 k

Ein anderes Problem?

Stell deine Frage

Willkommen bei der Mathelounge! Stell deine Frage einfach und kostenlos

x
Made by a lovely community